Piccioni milanesi!

Conteggi, probabilità, invarianti, logica, matematizzazione, ...
Rispondi
Avatar utente
enomis_costa88
Messaggi: 537
Iscritto il: 01 gen 1970, 01:00
Località: Brescia

Piccioni milanesi!

Messaggio da enomis_costa88 »

Questo era l'ultimo quesito della gara dell'unimi, penso sia estremamente importante e istruttivo averne visto almeno uno analogo una volta nella vita.

Dati n numeri interi dimostrare che posso sempre sceglierne alcuni la cui somma sia multipla di n.

Buon lavoro!
"Tu che lo vendi cosa ti compri di migliore?"

Membro dell' "Associazione non dimenticatevi dei nanetti! "
Membro dell'EATO.
Avatar utente
=Betta=
Messaggi: 22
Iscritto il: 09 mag 2006, 14:54
Località: Modena
Contatta:

Messaggio da =Betta= »

Scusami, ma non ho proprio capito... :( Ma gli n numeri devono essere consecutivi? E cos' è che dev'essere multiplo di n?
Avatar utente
edriv
Messaggi: 1638
Iscritto il: 16 feb 2006, 19:47
Località: Gradisca d'Isonzo
Contatta:

Messaggio da edriv »

Se non ha detto che sono consecutivi, vuol dire che possono essere anche non consecutivi...

Se dice che la somma di alcuni di quei numeri dev'essere multipla di n... sarà quella a dover essere multipla di n. Insomma, esiste un sottoinsieme di quell'insieme di interi tale che la somma di tutti i numeri che contiene è un multiplo di n.
Avatar utente
HiTLeuLeR
Messaggi: 1874
Iscritto il: 01 gen 1970, 01:00
Località: Reggio di Calabria

Messaggio da HiTLeuLeR »

Ma com'è che in queste gare made in Italy non si propongono mai (?) problemi originali? :?
uchiak
Messaggi: 56
Iscritto il: 17 dic 2006, 23:06

Messaggio da uchiak »

vediamo....
se divido a_1 , a_1 + a_2,...., a_1+....+a_n per n e trovo un resto zero il problema è risolto, sennò per il cassetto ce ne sono due con lo stesso resto e facendo la differenza risolvo il problema.
Lupacante
Messaggi: 42
Iscritto il: 31 gen 2008, 18:06

Messaggio da Lupacante »

uchiak ha scritto: ce ne sono due con lo stesso resto e facendo la differenza risolvo il problema.
facendo la differenza di cosa?
"se preceduto dalla propria citazione produce una falsità" se preceduto dalla propria citazione produce una falsità.
Avatar utente
EUCLA
Messaggi: 771
Iscritto il: 21 apr 2005, 19:20
Località: Prato

Messaggio da EUCLA »

Volendo riscriverla meglio...

Sia la successione di $ n $ numeri dati: $ a_1,a_2,...,a_n $

Sia $ \displaystyle B_t=\sum_{i=1}^{t}a_i $. I vari $ B $ formano ancora una successione di $ n $ numeri.

I resti modulo $ n $ sono $ n $, tra cui uno è lo $ 0 $

Caso 1: esiste $ B\equiv 0 \pmod n $

Caso 2: un tale $ B $ non esiste, allora due tra gli $ n \ B $ dovranno avere per pigeonhole la stessa congruenza modulo $ n $.

Ne faccio la differenza fra questi due che hanno la stessa congruenza, ottengo una cosa divisibile per $ n $ come richiesto.
Lupacante
Messaggi: 42
Iscritto il: 31 gen 2008, 18:06

Messaggio da Lupacante »

ma la consegna dice somma, non differenza
"se preceduto dalla propria citazione produce una falsità" se preceduto dalla propria citazione produce una falsità.
Avatar utente
EUCLA
Messaggi: 771
Iscritto il: 21 apr 2005, 19:20
Località: Prato

Messaggio da EUCLA »

Allora, la differenza che dicevamo la fai tra i $ B $

Ad esempio prendi $ B_2\equiv a \pmod n, B_{4}\equiv a \pmod n $

Per costruzione dei $ B $: $ B_{2}=a_1+a_2, B_{4}=a_1+a_2+a_3+a_4 $

La loro differenza è una somma (ed è congrua a 0 modulo n)!
Lupacante
Messaggi: 42
Iscritto il: 31 gen 2008, 18:06

Messaggio da Lupacante »

grazie eucla adesso capisco
"se preceduto dalla propria citazione produce una falsità" se preceduto dalla propria citazione produce una falsità.
Avatar utente
Cassa
Messaggi: 236
Iscritto il: 28 mar 2006, 21:48
Località: Genova

Messaggio da Cassa »

EUCLA ha scritto:Allora, la differenza che dicevamo la fai tra i $ B $

Ad esempio prendi $ B_2\equiv a \pmod n, B_{4}\equiv a \pmod n $

Per costruzione dei $ B $: $ B_{2}=a_1+a_2, B_{4}=a_1+a_2+a_3+a_4 $

La loro differenza è una somma (ed è congrua a 0 modulo n)!
Come fai ad affermare che ci saranno sempre 2 somme con la stessa congruenza ad n?
Avatar utente
EUCLA
Messaggi: 771
Iscritto il: 21 apr 2005, 19:20
Località: Prato

Messaggio da EUCLA »

Hai inizialmente $ n $ possibili congruenze.
Se però togli la possibilità che un $ B $ sia 0 modulo n, ne rimangono n-1.
Hai $ n \ B $, $ n-1 $ possibili congruenze → (piccioni)→ $ \exists B_a,B_b \vert B_a\equiv B_b \pmod n $
Avatar utente
Cassa
Messaggi: 236
Iscritto il: 28 mar 2006, 21:48
Località: Genova

Messaggio da Cassa »

Uh, vero! grazie! :D
Rispondi